User Avatar
enzonabiev863
Joined
Apr 2025
Subscription
Free
User Avatar

Thursday, Oct 29 2015

enzonabiev863

Please help understand tricky question!

If anyone has Practice exam 1 and can help me with a question that I am having trouble grasping I would greatly appreciate it. Its from Section 4 number 21 and its a must be true question. I can't wrap my mind around D being the correct answer because to me it is logically equivalent to answer choice E so I automatically eliminated both. How is "some" different from "many"?

User Avatar
enzonabiev863
Friday, Oct 23 2015

Thanks a lot for the help!!

User Avatar
enzonabiev863
Friday, Oct 23 2015

I guess I'm just a little confused on the comments regarding conditional answers weakening when it fails to consider an aspect. So for instance for #13 since the argument failed to consider that researchers may think that the journal in which the citation appears is not highly regarded by leading researchers then that would weaken the argument?

User Avatar
enzonabiev863
Friday, Oct 23 2015

Thank you for such a thorough explanation. I just want to clarify a few things that you mentioned. So when an answer choice say "If this....then this..." I shouldn't worry about actually determining if the sufficient condition occurred, but rather focus on whether the possibility of it occurring affecting the strength of the argument? Should I focus on the necessary part of the condition in determining if it addresses the gap of the argument? Sorry for asking so many questions

User Avatar
enzonabiev863
Thursday, Oct 22 2015

Oh sorry I didn't know that. The first one is from PT 4 section 1 #13. I was able to get it correct (D) by elimination but was hesitant due to not knowing if the sufficient condition had to be met. Does it not matter if we know that the researchers actually thought this way as long as it's a possibility then it weakens the argument? The next one is from PT 5 Section 3 #8 where i had trouble eliminating (E) even though (C) clearly weakens the argument. I guess the trouble I'm having is that I'm wasting time thinking about the conditional aspect of the answer and being unsure if it would actually affect the argument if its unclear that the sufficient condition has triggered. Is E incorrect because it doesn't matter that air pollution would increase if the total number of automobiles in use are increased as long as older automobiles are not the cause of the pollution? BTW Thanks a lot

User Avatar
enzonabiev863
Thursday, Oct 22 2015

Thanks for the response, Ya ill give 2 examples where a conditional answer is correct and another where its incorrect. Do you happen to have the early lsat such as PT 4 and PT 6? I can post the question if you don't.

Hi thanks in advance if anyone can help. How should I deal with conditional answer choices in strengthen and weaken questions. Does it matter if the sufficient condition is triggered in the stimulus or does it not matter? Sometimes I have trouble with these answer choices because I'm hesitant to choose them since the stimulus doesn't indicate that the sufficient condition has been met even though the necessary condition will either weaken or strengthen the argument depending on the question stem.

PrepTests ·
PT107.S2.P3.Q21
User Avatar
enzonabiev863
Wednesday, Jan 14 2015

Question 21 choice (A) and 19 lack clear reasoning for why answer choices are wrong so i will help those who are confused. For 21 (A) is wrong because there aren't supporting examples, but rather just results from an experiment which are different than examples. Examples are just a few among many others, while the passage only provides results from studies and experiments. The distinction between hypothesis and conclusion does not make it incorrect in my opinion because both need to have valid support.

Question 19 should be made into an abstract before looking at the choices. For example: He uses artificial device to determine whether animal behaves like you think it should-the animal should be attracted to the device. Also another crucial distinction is that a platypus clearly don't eat batteries so he can test specifically if the electricity was enough to attract the platypus. All of the answer choices are wrong for the same reason because its not surprising that the decoys could fool the animals. Whereas answer choice C, shows that animal would not go near the object unless it was attracted to the heat. Hope this helps those who were confused

I'm looking for anybody willing to meet up or at least discuss the lsat over skype. I usually study between 5-8 hours a day and have been practicing around low 170's, but have still have a lot of trouble with timing. I'm willing to help any of you with my strengths which is logical reasoning. I'm just looking for anyone who is very motivated to score very high and can make studying much more interactive by bouncing our minds and methods off each other. Anyways, if you're interested let me know at enzonabiev818@gmail.com. Thank you and wish you all the best of luck!

PrepTests ·
PT123.S4.P2.Q13
User Avatar
enzonabiev863
Wednesday, Dec 10 2014

Hi, I have a question regarding number 13 and answer choice (E). In passage A where does the author refer to modern day humans behavior in connection to the origins of music? His research evidence only talks about brain imaging studies and I don't see anything following that references their behavior. Rather I interpreted that the passage was arguing that since music and language share the same system for processing sound and language has adaptive value whereas music does not, that music originated on the "coattails" of language. I'm not understanding JY's understanding of why its correct because he refers to them both using research so are we supposed to infer from that fact, that they are actually using behavior of modern humans as evidence for the origin of music and language?

I'm having trouble understanding the logical difference between very similar sufficient assumption questions. The questions are from PT 36 SECTION 1 #18 and PT 63 SECTION 1 #10. I'm having trouble understanding why for PT 36 an answer choice (D) that satisfies the sufficient condition of the conclusion is correct but for PT 63 an answer choice (D) that satisfies the sufficient condition of the conclusion is incorrect. Thanks in advance if anyone can help!

PrepTests ·
PT109.S1.Q20
User Avatar
enzonabiev863
Monday, Mar 09 2015

My explanation for why C is the correct answer is that I think you must focus on the core argument structure first. The conclusion argues that the theory that painters describe their current diets in their paintings is wrong! The premises used to support the argument is that they would have to eat sea animals while traveling to the island. My initial thought was that the argument is assuming that the painters are painting while they are traveling at sea or that they didn't bring any food with them. With these thoughts I was able to eliminate all the answers except A, and C. I looked at A again and realized that this does weaken the argument because they didn't have to paint sea animals on the island because their diet consisted on land animals which allows the theory to still hold. I chose C because it only boosts the already stated premise that their were no sea animals painted. In saying that, I mean that it does not hurt the argument that the theory is incorrect because the author's argument still stands because even though they painted some land animals that doesn't mean that these paintings described their current diets. They could have painted them for their beauty, symbolism, etc. So the theory could still be completely wrong because there were no sea animals painted and C offers no alternative explanation for why there weren't any painted, it only boosts the stated premise.

PrepTests ·
PT142.S1.Q14
User Avatar
enzonabiev863
Tuesday, Oct 06 2015

This question seems harder than it should be. The indicator words allow you to say that the role is the conclusion, but the premises that follow don't support the idea that the risk of such a collision will occur in the future in my opinion. The premises discuss what will occur after the collision rather than support the conclusion that it is actually likely to occur at all. Can anyone help with my understanding?

User Avatar
enzonabiev863
Sunday, Dec 06 2015

I'm with you splitter! I missed only 6 total on both LR sections during my October practice test. On the real thing I had to guess on six because I ran out of time. This test is just so brutal! Just when you think you have a section down well it blows up in your face. Going in reading comp was my weakest section in terms of timing, but I finished that section on time during my test. LR was my strongest but for some reason they felt so much harder than recent exams. I know my nerves had a role but I felt so inefficient and was second guessing myself at times.

User Avatar
enzonabiev863
Sunday, Oct 04 2015

I don't know for me I rather have that then they taking it on a little college desk where you don't have space for anything other than the test and a pencil.

User Avatar
enzonabiev863
Sunday, Oct 04 2015

I don't know why you say that because I took my lsat in June at JFK and the tables were amazing as far as space. It was just one huge table instead little tiny tables for each student. How do you know there was no space if you didn't take the test?

PrepTests ·
PT143.S3.Q9
User Avatar
enzonabiev863
Saturday, Oct 03 2015

I thought this was a match the flaw argument when I read it I thought it committed an absence of evidence fallacy. Just because there wasn't any signs or evidence of life during excursions on the moon doesn't necessarily mean there hasn't ever been actual life. Maybe there was life on the moon when you didn't make an excursion? Is it wrong to view the stimulus this way?

PrepTests ·
PT143.S1.Q22
User Avatar
enzonabiev863
Friday, Oct 02 2015

I thought the flaw was that the whole doesn't equal the parts?

PrepTests ·
PT143.S1.Q17
User Avatar
enzonabiev863
Friday, Oct 02 2015

Isn't B just a mistaken negation?

PrepTests ·
PT143.S1.Q10
User Avatar
enzonabiev863
Friday, Oct 02 2015

So I thought E was just the contrapositive of the argument in the question. I thought the stimulus was arguing that if costs are greater than its benefits then its not justified. So when I read E I thought it was the contrapositive since its not designed to have net benefit for society then its too long or not justified. Is this the wrong way to view this?

Confirm action

Are you sure?